guia fisica 4 (2)

FÍSICA IV              GUIA DE PROBLEMAS  RESUELTOS                      ENRIQUE RUBIO  2011  FÍSICA IV         

Views 116 Downloads 141 File size 2MB

Report DMCA / Copyright

DOWNLOAD FILE

Recommend stories

Citation preview

FÍSICA IV             

GUIA DE PROBLEMAS  RESUELTOS     

           

    ENRIQUE RUBIO 

2011 

FÍSICA IV         

GUIA DE PROBLEMAS  RESUELTOS     

  Ley de Biot – Savart   Ley de Ampere  Ley de Faraday  Inductancia y energía  Magnetismo en la materia  Circuitos  Ondas electromagnéticas  Ecuaciones de Maxwell  Óptima geométrica   

    ENRIQUE RUBIO 

2011

2

Física IV. Guía de Problemas Resueltos  Prohibida la reproducción parcial o total de este material sin la autorización por escrito de los autores 

Amig@s A menos que sea absolutamente necesario, evita imprimir este material. Estarás ahorrando hojas y, en consecuencia, salvando árboles que dejarán de ser talados.

Cuidar al planeta es tu responsabilidad. No se la transfieras a otro. Puedes hacer mucho.

Gracias,

Enrique Rubio

2011 

Enrique Rubio  http://guiasdeejercicios.blogspot.com  Evita imprimir este documento si no es necesario. Si requieres imprimir una parte hazlo  en hojas de reciclaje. Estarás salvando al planeta 

3

Física IV. Guía de Problemas Resueltos  Prohibida la reproducción parcial o total de este material sin la autorización por escrito de los autores 

Índice Tabla de Ecuaciones......................................................................................................... 5 I. Capítulo 1: Ley de Biot – Savart .......................................................................... 10 II. Capítulo 2: Ley de Ampere.................................................................................... 33 III. Capítulo 3: Ley de Faraday ............................................................................... 71 IV. Capítulo 4: Inductancia y energía ................................................................. 107 V. Capítulo 5: Magnetismo en la materia ............................................................. 120 VI. Capítulo 6: Circuitos ......................................................................................... 131 VII. Capítulo 7: Ondas electromagnéticas.......................................................... 146 VIII. Capítulo 8: Ecuaciones de Maxwell.............................................................. 160 IX. Capítulo 9: Óptica geométrica ....................................................................... 191

2011 

Enrique Rubio  http://guiasdeejercicios.blogspot.com  Evita imprimir este documento si no es necesario. Si requieres imprimir una parte hazlo  en hojas de reciclaje. Estarás salvando al planeta 

4

Física IV. Guía de Problemas Resueltos  Prohibida la reproducción parcial o total de este material sin la autorización por escrito de los autores 

Tabla de Ecuaciones A continuación se presentan las ecuaciones fundamentales que empleará a lo largo de este curso. Asimismo, la mayoría de estas ecuaciones o fórmulas se utilizan en la resolución de los problemas presentes en esta Guía.

CANTIDAD FÍSICA

EXPRESIÓN MATEMÁTICA

Corriente eléctrica (flujo de carga en el

i=

tiempo):

r r i J = σE = A

Densidad de corriente eléctrica:

Ley de Gauss (para campo eléctrostático):

dq dt

r r 1 E ∫ ⋅ dA =

∫ ρdV =

ε 0 V (S )

S

1

ε0

Qencerrada ( S )

Potencial Eléctrico:

r r ∆V = − ∫ E( r ) ⋅ dr

Carga en un capacitor:

Q = CV

Corriente en un capacitor:

i=C

Capacitancia de un capacitor de placas

C=

paralelas:

movimiento:

r r r FM = idl × B

Fuerza Magnética sobre un alambre de

Ley de Biot – Savart:

Ley de Ampere:

r r µ 0 idl × rˆ dB = 4π r 2 r r r r B ⋅ d l = µ J ⋅ dA = µ 0 ienlazada 0∫ ∫ C

Flujo Magnético:

ε0 A

d r r r FM = qV × B

Fuerza magnética sobre una carga en

corriente I:

d V(t ) dt

S

r r Φ Magnético = ∫ B ⋅ dA S

2011 

Enrique Rubio  http://guiasdeejercicios.blogspot.com  Evita imprimir este documento si no es necesario. Si requieres imprimir una parte hazlo  en hojas de reciclaje. Estarás salvando al planeta 

5

Física IV. Guía de Problemas Resueltos  Prohibida la reproducción parcial o total de este material sin la autorización por escrito de los autores 

Ley de Faraday:

r

r

d Φ Magnético dt

ε inducida = ∫ Einducido ⋅ dl = − C

Inductancia o Coeficiente de Autoinducción:

NΦ Magnético

L=

Inductancia Mutua:

I

M=

NΦ Magnético I

Densidad de energía magnética:

B2 uB = 2µ 0

Densidad de energía eléctrica:

1 uE = ε 0 E 2 2

Energía almacenada en un inductor:

U=

1 2 LI 2

µ = µ 0 (1 + χ M )

Permeabilidad Magnética (en presencia de un material magnético): Vector inducción magnética:

r r H ∫ ⋅ dl = C

Campo magnético (en función de la intensidad

r r J ∫ ⋅ dA = ienlazada S

r r B = µH

de campo): Vector de Magnetización:

r 1 r r M = B−H

µ0

Velocidad de la luz:

Voltaje rms:

Corriente rms:

Reactancia capacitiva:

2011 

c=

1

µ 0ε 0

Vrms =

VMáx 2

I rms =

I Máx 2

XC =

1 ωC

Enrique Rubio  http://guiasdeejercicios.blogspot.com  Evita imprimir este documento si no es necesario. Si requieres imprimir una parte hazlo  en hojas de reciclaje. Estarás salvando al planeta 

6

Física IV. Guía de Problemas Resueltos  Prohibida la reproducción parcial o total de este material sin la autorización por escrito de los autores 

Reactancia inductiva:

X L = ωL

Factor de potencia:

f . p. = cosφ

Potencia promedio:

2 Ppromedio = I rmsVrms cosφ = I rms R

Relación entre la amplitud de los campos

Emáx =c Bmáx

eléctrico y magnético: Número de Onda:

k=

Frecuencia angular:

λ

=

2πf ω = c c

ω = 2πf =

Frecuencia:

2π T

f =

c

λ

r E = E máx cos(kx − ωt ) Eˆ r B = Bmáx cos(kx − ωt ) Bˆ r S P= c r S P=2 c r r ∆U P= = ∫ S ⋅ dA ∆t S

Campo eléctrico y magnético de una onda plana que se propaga en el eje X: Presión de radiación (superficie absorbente):

Presión de radiación (superficie no absorbente): Potencia (flujo de energía en el tiempo):

Momento lineal (superficie reflejante):

p=2

Momento lineal (cuerpo negro):

Corriente de desplazamiento:



∆U c

∆U c d r r = ε 0 ∫ E ⋅ dA dt S p=

iD = ε 0

d Φ Eléctrico dt

Ecuaciones de Maxwell (forma integral):

2011 

Enrique Rubio  http://guiasdeejercicios.blogspot.com  Evita imprimir este documento si no es necesario. Si requieres imprimir una parte hazlo  en hojas de reciclaje. Estarás salvando al planeta 

7

Física IV. Guía de Problemas Resueltos  Prohibida la reproducción parcial o total de este material sin la autorización por escrito de los autores 

1º Ley de Gauss (campo electrostático): 1º

r r 1 E ∫ ⋅ dA =

1

∫ ρdV = ε

ε 0 V (S )

S

2º Ley de Gauss (campo magnético):



Qencerrada ( S )

0

r r B ∫ ⋅ dA = 0 S

3º ε inducida

3º Ley de Faraday:

4º 4º Ley de Ampere – Maxwell:

r r d = ∫ Einducido ⋅ dl = − Φ Magnético dt C

r r r r d r r B ⋅ d l = µ J ⋅ d A + µ ε E ⋅ dA 0∫ 0 0 ∫C ∫ dt S S

Ecuaciones de Maxwell (forma integral):

1º Ley de Gauss (campo electrostático): 1º

r 1 ∇⋅E = ρ

ε0

2º Ley de Gauss (campo magnético): 2º

r ∇⋅B=0

3º Ley de Faraday: 3º

4º Ley de Ampere – Maxwell: 4º

Vector de Poynting:

r ∂ r ∇ × Einducido = − N B ∂t

r r ∂ r ∇ × B = µ 0 J + µ 0ε 0 E ∂t r r r E×B S=

µ0

Magnitud del Vector de Poynting:

2011 

r E2 B2 S = =c µ0 cµ 0

Enrique Rubio  http://guiasdeejercicios.blogspot.com  Evita imprimir este documento si no es necesario. Si requieres imprimir una parte hazlo  en hojas de reciclaje. Estarás salvando al planeta 

8

Física IV. Guía de Problemas Resueltos  Prohibida la reproducción parcial o total de este material sin la autorización por escrito de los autores 

Magnitud promedio del vector de Poynting:

r E2 B2 =c S promedio = 2cµ 0 2µ 0

Densidad de energía transportada por una

1 B2 2 = uE + uB = ε 0 E + 2 2µ 0

onda electromagnética:

u EB

u EB = ε 0 E 2 = Índice de refracción:

µ0

c V( luz −medio )

Ley de Snell:

n1 sin θ 1 = n2 sin θ 2

Ecuación de Reflexión de la luz:

1 1 2 1 + = = o i R f

Ecuación de Refracción de la luz:

n1 n2 n2 − n1 + = o i R

Magnificación (aumento lateral):

2011 

n=

B2

M =−

Enrique Rubio  http://guiasdeejercicios.blogspot.com  Evita imprimir este documento si no es necesario. Si requieres imprimir una parte hazlo  en hojas de reciclaje. Estarás salvando al planeta 

i o

9

Física IV. Guía de Problemas Resueltos  Prohibida la reproducción parcial o total de este material sin la autorización por escrito de los autores 

I.

Capítulo 1: Ley de Biot – Savart

1.1

Demuestre que el campo magnético producido por un alambre, que conduce una corriente I, sobre un punto P, en el mismo plano del alambre a una distancia x, se puede escribir como:

r µI B( P ) = 0 (cosθ 1 − cosθ 2 )(uˆT ) 4πx

P

θ2

I

θ1

A través de esta expresión, muestre que si el alambre es infinito, entonces se obtiene la expresión:

r µI B( P ) = 0 (uˆT ) 2πx Donde

uˆT

es el vector unitario tangencial a una circunferencia con centro en el eje del

alambre y radio x.

SOLUCIÓN Partiendo de la Ley de Biot - Savart, se tiene:

r (1) r µ 0 Idl × rˆ dB = . 4π r 2 En este caso, tomando en consideración que el eje del alambre coincide con el eje z, se tiene que:

r dl = dzkˆ Por lo tanto: 2011 

Enrique Rubio  http://guiasdeejercicios.blogspot.com  Evita imprimir este documento si no es necesario. Si requieres imprimir una parte hazlo  en hojas de reciclaje. Estarás salvando al planeta 

10

Física IV. Guía de Problemas Resueltos  Prohibida la reproducción parcial o total de este material sin la autorización por escrito de los autores 

r r dl × rˆ = dl rˆ sin θ (−iˆ) r dl × rˆ = (sin θ )dz (−iˆ) ( 2 )

z

( rˆ Es el vector unitario en la dirección de la línea que



r

une al diferencial de longitud del alambre, dl punto P)

ˆj



con el

Además,

I

v dl

x

θ

se

puede

escribir:

P tan θ = − x ⇒ z = − x cot θ ( 3) z

r

Figura 1.1.1

Asimismo, se tiene que:

sin θ =

x x ⇒r= ⇒ r 2 = x 2 csc 2 θ ( 4 ) r sin θ

Derivando la expresión (3) respecto al ángulo:

P

dz = x csc 2 θ ⇒ dz = x csc 2 θdθ ( 5) dθ Sustituyendo las ecuaciones (2), (4) y (5)

θ2

x

θ1

Z

I

en la

Figura 1.1.2

r dl

expresión (1), se tiene:

r µ 0 I sin θ ( x csc 2 θdθ ) r ˆ) ⇒ dB = µ 0 I sin θdθ (−iˆ) dB = ( − i 4π x 2 csc 2 θ 4πx Luego, este diferencial de longitud varía desde el ángulo

θ1

hasta el ángulo

θ2

(véase Figura

1.1.2). Entonces, integrando la expresión superior:

2011 

Enrique Rubio  http://guiasdeejercicios.blogspot.com  Evita imprimir este documento si no es necesario. Si requieres imprimir una parte hazlo  en hojas de reciclaje. Estarás salvando al planeta 

11

Física IV. Guía de Problemas Resueltos  Prohibida la reproducción parcial o total de este material sin la autorización por escrito de los autores 

r µ I (−iˆ) θ B= 0 (− cosθ θ ) 4πx 2

1

Finalmente, resulta:

r µ I B = 0 (cos θ1 − cos θ 2 )(−iˆ) 4πx

Este análisis se hizo para un alambre finito. Sin embargo, si se hace

θ1 → 0

y

θ 2 → π , se

estaría en el caso de un alambre muy largo (infinito). En este caso:

r µ I B = 0 (−iˆ) 2πx

2011 

Enrique Rubio  http://guiasdeejercicios.blogspot.com  Evita imprimir este documento si no es necesario. Si requieres imprimir una parte hazlo  en hojas de reciclaje. Estarás salvando al planeta 

12

Física IV. Guía de Problemas Resueltos  Prohibida la reproducción parcial o total de este material sin la autorización por escrito de los autores 

1.2

Se tienen dos conductores paralelos largos que

conducen corrientes en la misma

dirección. El conductor A conduce una corriente IA y se mantiene fijo en su posición. El conductor B conduce una corriente IB y se deja que se deslice hacia arriba y hacia abajo (paralelo a A), entre un conjunto de rieles no conductores. Si la densidad de masa por unidad de longitud del conductor B es

λ , ¿Para qué valor de la corriente IB se producirá un

equilibrio cuando la distancia entre los conductores es a?

IA

ˆj a



IB

SOLUCIÓN El alambre A, debido a que está conduciendo corriente, produce un campo magnético que afecta al alambre B y, en consecuencia, produce una fuerza magnética sobre éste. Ahora bien, dado que ambos alambres son muy largos (infinitos), entonces el campo magnético producido por el alambre A, será (véase el problema 1 en el que se demuestra esta expresión):

r µ I B = 0 A (−kˆ) 2πr (La dirección vectorial indicada es exclusiva para este caso. Si se generaliza, el

campo

magnético producido por un alambre sobre distintos puntos a la misma distancia del eje de éste será en dirección

uˆT , es decir, tangencial). r FM

En esta ecuación

IB

es la distancia que separa ambos alambres, en

este caso: a. Por lo tanto:

r mg Figura 1.2.1

2011 

r

r µI B A = 0 A (kˆ) (1) 2πa

Enrique Rubio  http://guiasdeejercicios.blogspot.com  Evita imprimir este documento si no es necesario. Si requieres imprimir una parte hazlo  en hojas de reciclaje. Estarás salvando al planeta 

13

Física IV. Guía de Problemas Resueltos  Prohibida la reproducción parcial o total de este material sin la autorización por escrito de los autores 

Entonces, haciendo un análisis de fuerzas sobre el alambre B (véase Figura 1.2.1) y considerando la condición de equilibrio, se tiene:

r r F ∑ alambre( B ) = maB = 0 (Esta es la condición de equilibrio)

r r FM + mB g = 0 ( 2 ) . Recordando que la fuerza magnética sobre un alambre, producida por un campo magnético externo, se puede escribir como:

r r ( 3) FM = Il × B Sustituyendo la expresión (3) en la ecuación (2), se tiene:

r r I B l × B A + mB g = 0 (Donde

r BA

indica que el campo magnético es producido por el alambre A).

Entonces:

I B l (−iˆ) × B A kˆ + mg (− ˆj ) = 0 ⇒ I B lB A ( ˆj ) + mg (− ˆj ) = 0 mg IB = lB A

( 4)

(Véase, en la dirección vectorial de la fuerza magnética, que cuando por dos

alambres

circulan

corrientes en la misma dirección, éstos se atraen)

2011 

Enrique Rubio  http://guiasdeejercicios.blogspot.com  Evita imprimir este documento si no es necesario. Si requieres imprimir una parte hazlo  en hojas de reciclaje. Estarás salvando al planeta 

14

Física IV. Guía de Problemas Resueltos  Prohibida la reproducción parcial o total de este material sin la autorización por escrito de los autores 

m Luego, se tiene que: λ = l

(5)

(Este valor corresponde a la densidad de masa por unidad de longitud)

Ahora, sustituyendo la expresión (5) y (1) en la ecuación (4), resulta, finalmente:

IB =

2πλag µ0 I A

Nota Importante: Recuerde que en la expresión para la fuerza magnética sobre un alambre de longitud l, escrita anteriormente, la corriente presente es la que circula por el alambre sobre el cual se está hallando la fuerza magnética, mientras que el campo, es el campo magnético externo que origina dicha fuerza. Es por ello que, en este caso en particular, cuando se está hallando la fuerza sobre el alambre B se toma en consideración la corriente que circula por este alambre (IB) y el campo externo que afecta a este alambre (el campo magnético producido por el alambre A en este caso).

2011 

Enrique Rubio  http://guiasdeejercicios.blogspot.com  Evita imprimir este documento si no es necesario. Si requieres imprimir una parte hazlo  en hojas de reciclaje. Estarás salvando al planeta 

15

Física IV. Guía de Problemas Resueltos  Prohibida la reproducción parcial o total de este material sin la autorización por escrito de los autores 

1.3

Dos alambres muy largos están suspendidos por cuerdas no conductoras de longitud a. Los alambres tienen densidad de masa por unidad de longitud

µ

y conducen corrientes de

igual magnitud y sentidos opuestos. Determine la corriente I para que los alambres formen un ángulo de 2φ, entre ellos, cuando están en equilibrio.

ϕ

ˆj

a



I

I

SOLUCIÓN Considere solo uno de los alambres para hacer el análisis de fuerzas (por medio de la segunda Ley de Newton). Asimismo, se establece la condición de equilibrio para dicho alambre. Entonces:

r T

r

∑F

ϕ

alambre

r FM r mg Figura 1.3.1

r = ma alambre = 0

(Esta es la condición de equilibrio)

r r FM + T + m g = 0

Aquí la fuerza magnética es la producida por un campo magnético externo sobre un alambre de longitud l. En este caso, el campo magnético externo es el producido por el segundo alambre. Asimismo, por la conclusión del ejercicio anterior, se sabe que cuando por dos alambres circulan corrientes en la misma dirección, los campos magnéticos de éstos producen fuerzas que inducen a que los alambres se atraigan, mientras que si las corrientes circulan en direcciones contrarias, entonces los alambres se repelen (que corresponde a este caso). Por lo tanto, tomando estas consideraciones, se tiene:

2011 

Enrique Rubio  http://guiasdeejercicios.blogspot.com  Evita imprimir este documento si no es necesario. Si requieres imprimir una parte hazlo  en hojas de reciclaje. Estarás salvando al planeta 

16

Física IV. Guía de Problemas Resueltos  Prohibida la reproducción parcial o total de este material sin la autorización por escrito de los autores 

(

)

FM (iˆ) + T cos ϕ ( ˆj ) + sin ϕ (−iˆ) + mg (− ˆj ) = 0 r (1) ∑ Fx = 0 ⇒ FM − T sin ϕ = 0 ( 2) r mg ∑ Fy = 0 ⇒ T cosϕ − mg = 0 ⇒ T = cosϕ

Sustituyendo la ecuación (2) en la ecuación (1):

FM = mg tan ϕ ( 3) Ahora bien, la fuerza magnética se puede escribir como:

r FM = IlB ( 4 ) Por otra parte, el campo magnético que produce dicha fuerza se puede escribir como:

r µ 0 I (5) B= 2πr Aquí, r es la distancia que separa a los dos alambres y se puede escribir de la siguiente manera:

r = 2a sin ϕ ( 6 ) Sustituyendo (6) en (5), resulta:

B=

µ0 I

(7)

4πa sin ϕ

Luego, combinando las expresiones (7) y (4) con la ecuación (3), se tiene:

2011 

Enrique Rubio  http://guiasdeejercicios.blogspot.com  Evita imprimir este documento si no es necesario. Si requieres imprimir una parte hazlo  en hojas de reciclaje. Estarás salvando al planeta 

17

Física IV. Guía de Problemas Resueltos  Prohibida la reproducción parcial o total de este material sin la autorización por escrito de los autores 

I 2 lµ 0 = mg tan ϕ . 4πa sin ϕ

Donde,

µ=

m l

Combinando ambas expresiones, resulta, finalmente:

I=

2011 

4πaµg sin 2 ϕ µ 0 cos ϕ

Enrique Rubio  http://guiasdeejercicios.blogspot.com  Evita imprimir este documento si no es necesario. Si requieres imprimir una parte hazlo  en hojas de reciclaje. Estarás salvando al planeta 

18

Física IV. Guía de Problemas Resueltos  Prohibida la reproducción parcial o total de este material sin la autorización por escrito de los autores 

1.4

Determinar el campo magnético en un punto P localizado a una distancia x de la esquina de un largo alambre doblado de la forma mostrada en la figura, sabiendo que por dicho alambre circula una corriente I.

P

x ˆj



I

SOLUCIÓN α2

α1 Figura 1.4.1

P Este problema se podrá resolver por medio de la Ley de Biot Savart. Por lo tanto:

r r µ 0 Idl × rˆ dB = 4π r 2 Sin embargo, la solución de esta expresión para un alambre finito

es:

B=

µ0 I (cosα 1 − cosα 2 ) (1) 4πr

(Véase el Problema 1.1 en el cual se demostró esta expresión) Ahora bien, para este caso en particular, se tiene que:

α1 ≈ 0

y

α2 =

π 2

(véase Figura 1.4.1 en la

cual se encuentra indicados los ángulos). Asimismo, la distancia perpendicular que separa al punto del alambre es:

r=x

Sustituyendo estas condiciones en la ecuación (1), resulta, finalmente: B =

2011 

µ0 I 4πx

Enrique Rubio  http://guiasdeejercicios.blogspot.com  Evita imprimir este documento si no es necesario. Si requieres imprimir una parte hazlo  en hojas de reciclaje. Estarás salvando al planeta 

19

Física IV. Guía de Problemas Resueltos  Prohibida la reproducción parcial o total de este material sin la autorización por escrito de los autores 

1.5

Se tiene un conductor, en forma de cuadrado de longitud de lado l, que conduce una corriente I. Calcule el campo magnético en el centro del cuadrado.

I P

l

SOLUCIÓN Cada uno de los cuatro segmentos que forman el cuadrado son alambres de longitud l cada uno. De la misma manera, se puede concluir que, debido a que cada uno de estos alambres se encuentra ubicado a la misma distancia del centro y cada uno conduce la misma corriente (ya que el cuadrado es simplemente un alambre continuo doblado en esta forma), el campo magnético total en el centro de dicho cuadrado es cuatro veces la contribución de uno de los alambres (en otras palabras, sumando las contribuciones magnéticas de cada uno de los alambres, dado que cada uno tiene la misma magnitud y dirección, el total será cuatro veces la magnitud del campo producido por uno solo de los alambres). Por lo tanto, por la Ley de Biot - Savart, se calcula el campo magnético de uno de los alambres:

r r µ 0 Idl × rˆ dB = 4π r 2

L P

α2

L

r α1

Recordando el resultado de esta ecuación para un alambre finito (véase problema 1.1), se tiene:

Figura 1.5.1

r µI Balambre = 0 (cosα 1 − cosα 2 )(− kˆ) (1) 4πr

En este caso, la distancia r es la distancia perpendicular que separa el alambre del punto en cuestión, en particular:

r =l/2

α 1 = 45º y α 2 = 135º 2011 

(siendo l el lado del cuadrado). Por otra parte, los ángulos serán:

(véase Figura 1.5.1 para observar los ángulos)

Enrique Rubio  http://guiasdeejercicios.blogspot.com  Evita imprimir este documento si no es necesario. Si requieres imprimir una parte hazlo  en hojas de reciclaje. Estarás salvando al planeta 

20

Física IV. Guía de Problemas Resueltos  Prohibida la reproducción parcial o total de este material sin la autorización por escrito de los autores 

Asimismo, la dirección del campo magnético se puede verificar, analíticamente, por medio del producto cruz presente en la ecuación de la Ley de Biot-Savart; y, además, por la regla de la mano derecha, ya que el campo magnético la cumple (colóquese el dedo pulgar en la dirección de la corriente, y gire el resto de los dedos de la mano hacia el punto sobre el cual se está hallando el campo magnético. De esta manera, se verifica la dirección obtenida analíticamente). Entonces:

r r BTotal ( P ) = 4 Balambre Sustituyendo el campo magnético del alambre, resulta, finalmente:

r µI BTotal ( P ) = 2 2 0 ( − kˆ) πl

2011 

Enrique Rubio  http://guiasdeejercicios.blogspot.com  Evita imprimir este documento si no es necesario. Si requieres imprimir una parte hazlo  en hojas de reciclaje. Estarás salvando al planeta 

21

Física IV. Guía de Problemas Resueltos  Prohibida la reproducción parcial o total de este material sin la autorización por escrito de los autores 

1.6

Un alambre recto infinitamente largo que conduce una corriente I1 está rodeado por un lazo, tal como se muestra en la figura. El lazo, con una longitud L y un radio R, conduce una corriente I2 y es concéntrico con el alambre. Calcule la fuerza magnética producida por el alambre recto sobre el lazo.



R ˆj

I2 I1

L



SOLUCIÓN Para hallar la fuerza magnética entre el alambre y el lazo se puede, en primer lugar, dividir el lazo en cuatro segmentos: los dos segmentos semicirculares y los segmentos rectos paralelos al alambre central. Ahora, se sabe que la fuerza magnética se puede escribir como:

r r r FM = il × B De esta forma, cuando el alambre es paralelo al campo magnético, entonces la fuerza sobre éste será cero. En este caso en particular, el campo magnético producido por el alambre que conduce la corriente I1 es:

r µ I B = 0 1 (uˆ t ) (1) 2πr (En el problema 1.1 se demostró que este es el campo magnético producido por un alambre infinito. Sin embargo, aquí se ha extendido el resultado no solo a un punto en particular P, sino a una serie de puntos ubicados a la misma distancia r del eje del alambre. De esta forma, la dirección del campo magnético se generaliza y por ello se coloca

2011 

uˆt )

Enrique Rubio  http://guiasdeejercicios.blogspot.com  Evita imprimir este documento si no es necesario. Si requieres imprimir una parte hazlo  en hojas de reciclaje. Estarás salvando al planeta 

22

Física IV. Guía de Problemas Resueltos  Prohibida la reproducción parcial o total de este material sin la autorización por escrito de los autores 

Entonces, se puede observar que la dirección del vector director del campo magnético es paralela a los alambres semicirculares que forman parte del lazo. Por lo tanto, la fuerza magnética sobre ellos será nula. Luego, solo resta calcular la fuerza magnética sobre los alambres rectos:

r r r FTotal = FM ( alambre1) + FM ( alambre 2 ) r r r r r FTotal = I 2l × B + I 2l × B ( 2 ) Sin embargo, se puede observar que para cada uno de estos alambres (debido a que cada uno de ellos son una serie de puntos ubicado a la misma distancia del alambre central), el campo magnético tiene una dirección en particular. Para el de la izquierda (alambre 1) el campo magnético esta dirigido en la dirección está dirigido en la dirección

− iˆ

iˆ , mientras que para el de la derecha (alambre 2) el campo

(recuerde que el campo magnético cumple la ya conocida regla

de la mano derecha. Coloque el dedo pulgar en la dirección de la corriente, y gire el resto de los dedos para obtener la dirección del campo)

Luego, sustituyendo la ecuación (1) en la ecuación (2), con las consideraciones anteriores, se tiene:

r µI µI FM = I 2 L(kˆ) × 0 1 (iˆ) + I 2 L(− kˆ) × 0 1 (−iˆ) 2πR 2πR Finalmente, resulta:

r µI IL FM = 0 2 1 ( j ) πR

2011 

Enrique Rubio  http://guiasdeejercicios.blogspot.com  Evita imprimir este documento si no es necesario. Si requieres imprimir una parte hazlo  en hojas de reciclaje. Estarás salvando al planeta 

23

Física IV. Guía de Problemas Resueltos  Prohibida la reproducción parcial o total de este material sin la autorización por escrito de los autores 

Se tiene un alambre que se ha doblado en la forma de un cuadrado, de lado L, como se

1.7

muestra en la figura. Si la corriente que circula por el alambre es I, muestre que el campo magnético en un punto P, ubicado a una distancia d, perpendicular al centro del cuadrado se puede escribir como:

Y L

r B P P = X

I

d

Z

µ 0 IL2 ⎛ ⎛L⎞ ⎞ 2π ⎜⎜ d 2 + ⎜ ⎟ ⎟⎟ ⎝2⎠ ⎠ ⎝ 2

3 2



SOLUCIÓN Estudiando la simetría del problema, y comprobándola a través de la regla de la mano derecha, se puede determinar que las componentes del campo magnético se cancelarán tanto en el eje Z como en el eje Y. ˆj

inferior (los que son paralelos al eje Z), las

r B1 y

α α

Explicación: Si se analiza el alambre superior e iˆ

r BAlambre( Superior )

r

α

r r B1 x + B2 x

r r B B2 y Alambre (inf erior )

componentes del campo magnético producido por cada uno de dichos alambres en el eje Y se anularán entre sí, mientras que las componentes del eje X se sumarán. Por otra parte, si se analizan los alambres laterales (los que están paralelos al eje Y), se podrá observar como las componentes del campo magnético en el eje Z también se anularán entre sí, mientras que las componentes en el eje X se suman.

Figura 1.7.1 Vista Lateral

Para poder demostrar esto, se analizará el campo magnético de los alambres superior e inferior.

2011 

Enrique Rubio  http://guiasdeejercicios.blogspot.com  Evita imprimir este documento si no es necesario. Si requieres imprimir una parte hazlo  en hojas de reciclaje. Estarás salvando al planeta 

24

Física IV. Guía de Problemas Resueltos  Prohibida la reproducción parcial o total de este material sin la autorización por escrito de los autores 

Descripción de la figura. En la figura 1.7.1 se tiene una vista lateral del sistema mostrado en la figura del enunciado del problema. Los alambres superior e inferior no se ven, sin embargo, están representados por los puntos superior e inferior unidos por el lado del cuadrado que si se puede apreciar. Ahora bien, los círculos punteados representan el campo magnético producido por cada uno de ellos (el campo magnético producido por un alambre es tangencial a un círculo centrado en dicho alambre). Entonces, para el punto P, en particular, el campo magnético producido por los alambres superior e inferior tiene la dirección mostrada en la figura, para cada uno de ellos.

En consecuencia, si se proyecta el campo magnético de cada uno de los alambres en el sistema coordenado de referencia, entonces, se tendrá que las componentes en el eje Y están en sentidos opuestos, mientras que las componentes en el eje X están en el mismo sentido. Por lo tanto, sabiendo que el módulo del campo magnético superior e inferior es el mismo, por tratarse de alambres de la misma longitud y por los cuales circula la misma corriente, las componentes del eje Y se anularán, mientras que las del eje X se sumarán.

Ahora bien, si se hace el mismo análisis para los alambres laterales, se podrá concluir que las componentes en el eje Z se anularán, mientras que las componentes en el eje X se sumarán. De esta forma, el campo magnético total se podrá escribir como:

(Donde

r Bx

r r (1) BTotal = 4 Bx

es la componente en el eje X de cualquiera de los alambres, debido a que dicha

componente para los cuatro alambres es la misma)

Por la Ley de Biot – Savart, se tiene:

r ( 2) r µ 0 Idl × rˆ dB = 4π r 2

Para conseguir la componente en el eje X basta con utilizar las relaciones trigonométricas:

r r dBx = dB cosα ( 3)

(Donde

α

es el ángulo que corresponde a la proyección sobre el eje X)

Sustituyendo (2) en (3), resulta (esto corresponde a la realización del producto vectorial o producto cruz):

2011 

Enrique Rubio  http://guiasdeejercicios.blogspot.com  Evita imprimir este documento si no es necesario. Si requieres imprimir una parte hazlo  en hojas de reciclaje. Estarás salvando al planeta 

25

Física IV. Guía de Problemas Resueltos  Prohibida la reproducción parcial o total de este material sin la autorización por escrito de los autores 

r µ Idl dBx = 0 2 cosα ( 4 ) 4π r

Asimismo, se puede deducir lo siguiente:

L/2 cosα = r

Luego, por Teorema de Pitágoras, se tiene:

( 5)

⎛ ⎛L⎞ r = ⎜⎜ d 2 + ⎜ ⎟ ⎝2⎠ ⎝

Sustituyendo las igualdades (6) y (5) en la expresión (4):

2

⎞ ⎟ ⎟ ⎠

1 (6) 2

r µ L dBx = 0 8π

Idl ⎛ 2 ⎛ L ⎞2 ⎞ ⎜d + ⎜ ⎟ ⎟ ⎜ ⎝ 2 ⎠ ⎟⎠ ⎝

3 2



En esta igualdad, los valores de d y L son constantes, por lo tanto, integrando:

r r = d B B ∫ x x=

µ 0 LI

⎛ 2 ⎛ L ⎞2 ⎞ 8π ⎜⎜ d + ⎜ ⎟ ⎟⎟ ⎝2⎠ ⎠ ⎝ r µ 0 L2 I Bx = iˆ ( 7 ) 3 ⎛ 2 ⎛ L ⎞2 ⎞ 2 8π ⎜⎜ d + ⎜ ⎟ ⎟⎟ ⎝2⎠ ⎠ ⎝

L

∫ dl (iˆ)

3 2 0

Finalmente, sustituyendo (7) en (1), resulta:

2011 

Enrique Rubio  http://guiasdeejercicios.blogspot.com  Evita imprimir este documento si no es necesario. Si requieres imprimir una parte hazlo  en hojas de reciclaje. Estarás salvando al planeta 

26

Física IV. Guía de Problemas Resueltos  Prohibida la reproducción parcial o total de este material sin la autorización por escrito de los autores 

r BTotal =

2011 

µ 0 L2 I ⎛ ⎛L⎞ 2π ⎜ d 2 + ⎜ ⎟ ⎜ ⎝2⎠ ⎝

2

⎞ ⎟ ⎟ ⎠

3 2



Enrique Rubio  http://guiasdeejercicios.blogspot.com  Evita imprimir este documento si no es necesario. Si requieres imprimir una parte hazlo  en hojas de reciclaje. Estarás salvando al planeta 

27

Física IV. Guía de Problemas Resueltos  Prohibida la reproducción parcial o total de este material sin la autorización por escrito de los autores 

Dos bobinas circulares de radio R se encuentran paralelas. Sus centros están separados

1.8

una distancia R y están ubicados sobre el mismo eje. Una corriente, I, circula en direcciones opuestas por cada bobina. Demuestre que el campo magnético a una distancia d del centro de una de las bobinas, está dado por:

Y

r µ 0 IR 2 ⎛⎜ 1 1 B= − 3 3 2 ⎜⎜ (R 2 + d 2 )2 2 2 2 (( R − d ) + R ) ⎝

R

d

⎞ ⎟ˆ ⎟⎟i ⎠

Anillo (1)

X Anillo (2)

SOLUCIÓN En primer lugar, se considerará que el origen del sistema de coordenadas se encuentra en el centro del anillo 1. Entonces, se calculará el campo magnético producido por cada uno de los anillos en el punto en cuestión.

Anillo (1): (Para la descripción de esta figura, vaya a la descripción de la

ˆj

figura del problema anterior)



r B1

Se puede observar, en la figura, el campo magnético producido por la parte superior e inferior del anillo (tal como el en el problema 1.7. No obstante, en este problema estas partes no son alambres sino diferenciales de longitud). Nuevamente, las componentes en el eje Y se anularán, mientras que las componentes en el eje X se

r B1

d Figura 1.8.1 Vista Lateral

2011 

sumarán. Entonces, si se considera que el campo magnético producido por el anillo 1 forma un ángulo que:

r r (1) B1 = 2 B1x

α

con el eje X, se tendrá

Enrique Rubio  http://guiasdeejercicios.blogspot.com  Evita imprimir este documento si no es necesario. Si requieres imprimir una parte hazlo  en hojas de reciclaje. Estarás salvando al planeta 

28

Física IV. Guía de Problemas Resueltos  Prohibida la reproducción parcial o total de este material sin la autorización por escrito de los autores 

(En este caso se multiplica por el factor 2, ya que cada diferencial de longitud tiene sólo un elemento diferencial, diametralmente opuesto. En el caso del problema anterior, se tenían 4 segmentos del mismo alambre. Aquí, se tiene dos segmentos semicirculares, el inferior y el superior)

Ahora, por la Ley de Biot – Savart, resulta:

r ( 2) r µ 0 Idl × rˆ dB = 4π r 2

Luego, para conseguir la componente en el eje X basta con utilizar las relaciones trigonométricas:

r r dB1 x = dB cosα ( 3)

Sustituyendo (2) en (3), será:

r µ Idl dB1x = 0 2 cosαiˆ ( 4 ) 4π r

De la gráfica, se puede deducir que:

R cosα = r

( 5)

Donde r es la distancia que separa a los diferenciales de longitud del anillo del punto P y, por lo tanto, se puede escribir como:

r = (R + d 2

)

1 (6) 2 2

Asimismo, el diferencial de longitud en el caso de un anillo (o círculo), se puede escribir como:

dl = Rdφ ( 7 ) (Donde

φ

es el ángulo polar).

Sustituyendo las igualdades (5), (6) y (7) en la expresión (4), resulta:

2011 

Enrique Rubio  http://guiasdeejercicios.blogspot.com  Evita imprimir este documento si no es necesario. Si requieres imprimir una parte hazlo  en hojas de reciclaje. Estarás salvando al planeta 

29

Física IV. Guía de Problemas Resueltos  Prohibida la reproducción parcial o total de este material sin la autorización por escrito de los autores 

r µ 0 IR 2 dφ ˆ dB1 x = i 3 4π (R 2 + d 2 )2 Es importante notar que, en este caso, el ángulo polar se integrará solo en la mitad del ángulo que forma el círculo (que es 2 π ). Esto se debe a que, al considerar el efecto de la parte superior e inferior en la misma gráfica y concluyendo que el efecto en el eje Y se anula, mientras que el efecto en el eje X se suma, entonces ya se está incluyendo en la misma expresión ambas mitades de las bobinas, por lo que solo bastará integrar una mitad.

De esta forma:

µ0 R 2 I

r r d B B = ∫ 1x 1x =

4π (R 2 + d

π

)

3 2 2



0

r dφiˆ ⇒ B1x =

µ0 R 2 I 4(R 2 + d

)

3 2 2

iˆ ( 8)

Sustituyendo (8) en (1), resulta:

r B1 =

µ0 R 2 I 2(R 2 + d

)

3 2 2

iˆ ( 9 )

Anillo (2):

Y r B2

En este caso, ocurrirá exactamente lo mismo que en el

ˆj

caso del anillo 1. Las componentes del campo magnético



en el eje Y se anularán, mientras que las componentes en el eje X se sumarán. Si se hace el análisis completo, surgirá un resultado similar al anterior. No obstante, la

r B2

X R−d

única diferencia es el ángulo que forma la línea del campo magnético con el eje X y, también, que la dirección del campo total es

Figura 1.8.2 Vista Lateral

− iˆ . Por lo tanto, con la expresión (4) del

anillo anterior (y colocándole su respectiva dirección), se tiene:

2011 

Enrique Rubio  http://guiasdeejercicios.blogspot.com  Evita imprimir este documento si no es necesario. Si requieres imprimir una parte hazlo  en hojas de reciclaje. Estarás salvando al planeta 

30

Física IV. Guía de Problemas Resueltos  Prohibida la reproducción parcial o total de este material sin la autorización por escrito de los autores 

r µ Idl dB2 x = 0 2 cosα 1 (−iˆ) (10 ) 4π r

En este caso:

Donde:

R cosα 1 = r

(11)

r = (( R − d ) + R 2

)

1 (12 ) 2 2

Sustituyendo las igualdades (7), (11) y (12) en la expresión (10), se tiene:

r r = d B B ∫ 2x 2x = r B2 x =

µ0 R 2 I

π

4π (( R − d ) + R 2

µ0 R 2 I 4(( R − d ) 2 + R

)

3 2 2

)

3 2 2

∫ dφ (−iˆ) 0

(−iˆ) (13)

(Véase que los límites de integración del ángulo polar son los mismos que en el caso anterior. La razón es, obviamente, la misma) Ahora, el campo magnético total del anillo 2, será:

r r B2 = 2 B2 x =

µ0 R 2 I 2(( R − d ) 2 + R

)

3 2 2

(−iˆ) (14 )

Entonces, el campo magnético total en el punto P se puede escribir como:

r r r (15 ) BTotal = B1 + B2

Finalmente, sustituyendo en la ecuación (15) las expresiones (9) y (14):

2011 

Enrique Rubio  http://guiasdeejercicios.blogspot.com  Evita imprimir este documento si no es necesario. Si requieres imprimir una parte hazlo  en hojas de reciclaje. Estarás salvando al planeta 

31

Física IV. Guía de Problemas Resueltos  Prohibida la reproducción parcial o total de este material sin la autorización por escrito de los autores 

r µ R2I BTotal = 0 2

2011 

⎛ 1 ⎜ ⎜ ⎜ R2 + d 2 ⎝

(



1

) (( R − d ) 3 2

2

+ R2

)

3 2

⎞ ⎟ ˆ ⎟ (i ) ⎟ ⎠

Enrique Rubio  http://guiasdeejercicios.blogspot.com  Evita imprimir este documento si no es necesario. Si requieres imprimir una parte hazlo  en hojas de reciclaje. Estarás salvando al planeta 

32

Física IV. Guía de Problemas Resueltos  Prohibida la reproducción parcial o total de este material sin la autorización por escrito de los autores 

II.

Capítulo 2: Ley de Ampere

2.1

Un cilindro macizo de radio a, lleva una densidad de corriente que varía con la distancia radial al eje, r, de acuerdo a la relación:

r 2I 0 ⎛ r2 ⎞ J ( r ) = 2 ⎜1 − 2 ⎟ πa ⎝ a ⎠

donde I0 es una constante.

2.1.1

¿Cuál es la corriente total que pasa por su sección transversal?

2.1.2

Determine el campo magnético en todo el espacio.

a

r J

SOLUCIÓN 2.1.1

Para hallar la corriente total que atraviesa el cilindro mostrado, se tiene:

(1) r r r r 2I o r2 ˆ r di = J ⋅ dA ⇒ iTotal = ∫ J ⋅ dA ⇒ iTotal = ∫ 2 (1 − 2 )k ⋅ dA A A πa a

(Véase que, como se quiere la corriente total que circula por el cilindro, entonces se integra toda el área transversal de dicho cilindro)

Ahora, el diferencial de área (transversal) de un cilindro se puede escribir como:

r dA = 2πrdrkˆ ( 2 ) (En este caso, el vector director de este diferencial está sobre el eje Z) 2011 

Enrique Rubio  http://guiasdeejercicios.blogspot.com  Evita imprimir este documento si no es necesario. Si requieres imprimir una parte hazlo  en hojas de reciclaje. Estarás salvando al planeta 

33

Física IV. Guía de Problemas Resueltos  Prohibida la reproducción parcial o total de este material sin la autorización por escrito de los autores 

Sustituyendo la ecuación (2) en la (1), se tiene:

iTotal

r2 ˆ r3 2Io 4Io a ˆ = 2 ∫ (1 − 2 )k ⋅ 2π rdrk ⇒ iTotal = 2 ∫ (r − 2 )dr a a o a πa A

(Se debe recordar que el producto punto, o escalar, de dos vectores unitarios en la misma dirección es 1,

iTotal

kˆ ⋅ kˆ = 1 )

4I ⎛ 1 1 = 2o ⎜⎜ r 2 − 2 r 4 a ⎝2 4a

⎞ ⎟ ⎟ 0 ⎠ a

Finalmente, resulta:

iTotal = I o 2.1.2

El campo magnético, en el caso de los cilindros, depende de la distancia del punto sobre el que se está hallando el campo al eje del cilindro. Por lo tanto, se deberá separar el análisis de este problema en dos regiones, dentro y fuera del cilindro.

Ahora, a través de la Ley de Ampere, se tiene:

r r r r B ⋅ d l = µ J ⋅ dA = µ 0 iEnlazada ( C ) 0∫ ∫

C ( A)

A

Nota Importante: se debe recordar que para las trayectorias utilizadas en la integral de la Ley de Ampere el campo magnético debe ser constante. En otras palabras, para todos los puntos ubicados a la misma distancia del centro del cilindro, el campo magnético debe tener la misma magnitud (más no la misma dirección).

2011 

Enrique Rubio  http://guiasdeejercicios.blogspot.com  Evita imprimir este documento si no es necesario. Si requieres imprimir una parte hazlo  en hojas de reciclaje. Estarás salvando al planeta 

34

Física IV. Guía de Problemas Resueltos  Prohibida la reproducción parcial o total de este material sin la autorización por escrito de los autores 

Ahora bien, para saber la dirección en la que está el campo magnético (que será la misma que la del diferencial de trayectoria o de longitud), se empleará la regla de la mano derecha, es decir, se coloca el dedo pulgar en la dirección de la corriente y girando el resto de los dedos se obtendrá dicha dirección.

En este problema, en particular, (y, en general, en todos los problemas con simetría cilíndrica), la dirección del campo será tangencial a la trayectoria utilizada (véase Figura 2.1.1). Por lo tanto, denotando este vector unitario como

uˆt a

Trayectori a Amperiana

r

(unitario tangencial), se tendrá:

r J

r dA ut

Z

Cilindro

Figura 2.1.1

1º Para

r < a : (se tomará una trayectoria cuyo radio es menor que el radio del cilindro)

r r r r (1) B ⋅ d l = µ J 0 ∫ ∫ ⋅ dA C

A

Sustituyendo la expresión de la densidad de la corriente en la ecuación anterior:

2I o r2 ˆ ∫C B(uˆt ) ⋅ dl (uˆt ) = µ 0 ∫A πa 2 (1 − a 2 )k ⋅ 2πrdrk 4µ I B ∫ dl = 02 o a C

4µ 0 I o ⎛ 1 2 r3 ⎜ r − 12 r4 r dr B r ( ) ( 2 ) − ⇒ = π 2 2 ∫0 a a ⎜⎝ 2 4a r

⎞ ⎟ ⎟ 0 ⎠ r

(Vea que la Ley de Ampere, solo se proporciona la magnitud del campo magnético. Sin embargo, después de aplicar la regla de la mano derecha, ya se sabe cual es su dirección. Por otra parte, la

2011 

Enrique Rubio  http://guiasdeejercicios.blogspot.com  Evita imprimir este documento si no es necesario. Si requieres imprimir una parte hazlo  en hojas de reciclaje. Estarás salvando al planeta 

35

Física IV. Guía de Problemas Resueltos  Prohibida la reproducción parcial o total de este material sin la autorización por escrito de los autores 

integral del diferencial de longitud a lo largo de toda la trayectoria es igual a la longitud de esa trayectoria. Recuerde, pues, que la longitud de una trayectoria circular es

2π ∗ radio )

Finalmente, evaluando la expresión anterior y colocando la dirección del campo, resulta:

r r2 µo I o B( r < a ) = r (1 − 2 )(uˆt ) 2a πa 2

2º Para

r ≥ a : (se tomará una trayectoria cuyo radio es mayor al del cilindro)

r r r r B ⋅ d l = µ J ⋅ dA 0∫ ∫ C

A

La diferencia entre esta región y la anterior, es que la corriente enlazada por la trayectoria será la corriente total que pasa por el cilindro. Por lo tanto, la integral:

r r J ∫ ⋅ dA

es igual a la corriente total

A

del cilindro, es decir

I o . Por lo tanto, resultará que el campo magnético fuera del cilindro es:

r µ I B( r ≥ a ) = 0 0 (uˆt ) 2πr

2011 

Enrique Rubio  http://guiasdeejercicios.blogspot.com  Evita imprimir este documento si no es necesario. Si requieres imprimir una parte hazlo  en hojas de reciclaje. Estarás salvando al planeta 

36

Física IV. Guía de Problemas Resueltos  Prohibida la reproducción parcial o total de este material sin la autorización por escrito de los autores 

Un cilindro largo de radio a tiene dos cavidades cilíndricas de radio a/2 a lo largo de toda su

2.2

longitud. Una corriente I circula en la dirección del eje Z, distribuida uniformemente en toda su sección transversal. Determine la magnitud y dirección del campo magnético en un punto del eje X a distancia r del centro del cilindro.

I

a/2

a

X r

SOLUCIÓN Para hallar el campo magnético con esta configuración, se debe aplicar el Principio de Superposición. Para ello se seguirán los siguientes pasos: ƒ

Se calculará el campo magnético que produce el cilindro grande, sobre el punto, como si no |tuviese ninguna cavidad. Pero, se debe hacer notar que la corriente que se tomará en consideración para hallar este campo no es la que pasa por la sección transversal del cilindro (que es el dato en el problema), sino la total que circularía si no existiesen cavidades (se hará a través de una regla de 3, ya que la corriente es uniforme

ƒ

Se calculará el campo magnético que producen cada una de las cavidades sobre el punto. Pero, así como en el caso anterior, se debe hallar la corriente que circula por dichas cavidades empleando una regla de tres (que en realidad no existe, pero para efectos del Principio de Superposición, la se tiene que considerar).

ƒ

Ahora, se restarán los tres efectos (ya que se ha hallado el campo magnético de unas cavidades por las cuales no circula corriente, por lo tanto su efecto no existirá. Es por esta razón, que se debe restárselo al efecto del cilindro grande sin cavidades).

De esta forma, se tendrá que el campo magnético total sobre el punto, se calculará con la expresión:

2011 

Enrique Rubio  http://guiasdeejercicios.blogspot.com  Evita imprimir este documento si no es necesario. Si requieres imprimir una parte hazlo  en hojas de reciclaje. Estarás salvando al planeta 

37

Física IV. Guía de Problemas Resueltos  Prohibida la reproducción parcial o total de este material sin la autorización por escrito de los autores 

r r r r (1) BTotal = BCilindro − BCavidad (1) − BCavidad ( 2 ) 1º Cilindro grande:

r r r r B ⋅ d l = µ J ⋅ dA . 0∫ ∫ C

A

Véase que el punto está afuera de todos los cilindros, por lo tanto, para las tres trayectorias amperianas que se aplicarán, se tomará en cuenta la corriente total que pasa por cada uno de los cilindros. Ahora, después de hacer una regla de tres, resulta:

I Cilindro = 2 I Entonces:

r r µI µI BCilindro = 0 Cilindro (uˆ t ) ⇒ BCilindro = 0 (uˆ t ) ( 2 ) πr 2πr 2º Cavidad izquierda:

A través de una regla de tres, se puede demostrar que la corriente, equivalente, que circularía por cada una de las cavidades es:

I Cavidades =

1 I 2

También, para la cavidad izquierda, la distancia entre ésta y el punto, será:

r + a/2 Por lo tanto:

2011 

Enrique Rubio  http://guiasdeejercicios.blogspot.com  Evita imprimir este documento si no es necesario. Si requieres imprimir una parte hazlo  en hojas de reciclaje. Estarás salvando al planeta 

38

Física IV. Guía de Problemas Resueltos  Prohibida la reproducción parcial o total de este material sin la autorización por escrito de los autores 

r r µ I µ0 I (uˆ t ) ( 3) BCavidad (1) = o Cavidad (uˆ t ) ⇒ BCavidad (1) = a a 2π (r + ) 4π (r + ) 2 2 3º Cavidad derecha:

La única diferencia con la anterior, es la distancia del punto al eje de la cavidad, que en este caso será:

r − a/2 r r µ I µo I BCavidad ( 2) = o Cavidad (uˆ t ) ⇒ BCavidad ( 2) = (uˆ t ) ( 4 ) a a 2π (r − ) 4π (r − ) 2 2 Finalmente, sustituyendo las expresiones (2), (3) y (4) en la ecuación (1), resultará:

r µ I BTotal = 0 (uˆ t ) − πr

µ0 I a 4π (r + ) 2

(uˆ t ) −

µ0 I a 4π (r − ) 2

(uˆ t )

Simplificando la expresión anterior, se obtendrá:

r µ I BTotal = 0 πr

2011 

⎛ 2r 2 − a 2 ⎞ ⎜⎜ 2 ⎟(uˆ ) 2 ⎟ t ⎝ 4r − a ⎠

Enrique Rubio  http://guiasdeejercicios.blogspot.com  Evita imprimir este documento si no es necesario. Si requieres imprimir una parte hazlo  en hojas de reciclaje. Estarás salvando al planeta 

39

Física IV. Guía de Problemas Resueltos  Prohibida la reproducción parcial o total de este material sin la autorización por escrito de los autores 

2.3

Un cable coaxial se forma redondeando un conductor cilíndrico sólido de radio R1 con un cilindro coaxial de radio interno R2 y externo R3. La corriente que circula por el cable interior regresa por la capa exterior. Determine el campo magnético en puntos en las distintas regiones dentro y fuera del cable. Hacer el gráfico de

r B

en función de r. Suponer que la

densidad de corriente es uniforme.

I R2

R1 R3

I SOLUCIÓN

En este caso, se tendrá que separar el análisis del problema en 4 regiones. Dichas regiones son:

1.

r < R1

2.

R1 ≤ r < R2

3.

R2 ≤ r < R3

4.

r ≥ R3 Ahora, empleando la Ley de Ampere a cada una de las regiones denotadas anteriormente, se tendrá:

R1

r

r r r r (1) B ⋅ d l = µ J 0 ∫ ∫ ⋅ dA C

A

Trayectori a Amperiana

Figura 2.3.1

1º Para

r < R1

(véase Figura 2.3.1):

Como la corriente I que pasa por el cilindro interior, se distribuye uniformemente en toda su sección transversal, entonces, se podrá escribir: 2011 

Enrique Rubio  http://guiasdeejercicios.blogspot.com  Evita imprimir este documento si no es necesario. Si requieres imprimir una parte hazlo  en hojas de reciclaje. Estarás salvando al planeta 

40

Física IV. Guía de Problemas Resueltos  Prohibida la reproducción parcial o total de este material sin la autorización por escrito de los autores 

r r r r dI = J ⋅ dA ⇒ I Total = ∫ J ⋅ dA ⇒ I Total = J ∫ dA A

r I ⇒ J = Total2 πR1

A

( 2)

Sustituyendo la expresión (2) en la (1), se tiene:

B ∫ dl = µ 0 C

I Total I Total dA ⇒ B ( 2 π r ) = µ 2πrdr 0 2 2 πR1 ∫S πR1 ∫S r

r 2I 2I 1 B(2πr ) = µ 0 Total rdr ⇒ B (2πr ) = µ 0 Total ( r2 ) 2 2 ∫ R1 2 0 R1 0

(Vea que, en este caso, la región de integración es el área transversal definida por el radio de la trayectoria amperiana)

Evaluando esta expresión y colocando la dirección del campo magnético, resulta:

r µ Ir B( r < R1 ) = o 2 (uˆt ) 2πR1

R2

2º Para

r

R1

R1 ≤ r < R2

(véase Figura 2.3.2):

En esta región, la trayectoria amperiana que se está utilizando enlaza por completo al cilindro interior de radio R1, por lo tanto, la corriente enlazada será la corriente total que pasa a través del área transversal Trayectori a Amperiana Figura 2.3.2

2011 

de este cilindro. De esta manera, el campo magnético resultará:

r µ I B( R1 ≤ r < R2 ) = o (uˆt ) 2πr

Enrique Rubio  http://guiasdeejercicios.blogspot.com  Evita imprimir este documento si no es necesario. Si requieres imprimir una parte hazlo  en hojas de reciclaje. Estarás salvando al planeta 

41

Física IV. Guía de Problemas Resueltos  Prohibida la reproducción parcial o total de este material sin la autorización por escrito de los autores 

3º Para

R2 ≤ r < R3

(véase Figura 2.3.3):

Aquí, se utilizará el Principio de Superposición. Es decir, para ecampo magnético se considerará el efecto del cilindro interior, de

R3 R 1 r R2

radio R1, y el de la región entre la concha cilíndrica interior, de radio R2, y la exterior, de radio R3, enlazadas por la trayectoria amperiana. Asimismo, se debe hacer notar que las corrientes en el cilindro Trayectori a

interior y en la concha tienen sentidos opuestos, por lo tanto, los

Amperiana

campos magnéticos de cada uno de ellos tendrám sentidos

Figura 2.3.3

opuestos.

Entonces, la densidad de corriente en el caso de la concha cilíndrica enlazada, se podrá escribir como:

r r di = J ⋅ dA ⇒ I Total ( concha ) =

r r J ∫ ⋅ dA ⇒ I Total (concha ) = J

A ( concha )

r R I Total ( concha ) = J ∫ 2πrdr ⇒ J = 1

R2

∫ dA

A ( concha )

( 3)

I Total ( concha )

π ( R3 − R2 ) 2

2

Sustituyendo la ecuación (3) en la ecuación (1):

Bconcha ∫ dl = µ 0 C

I Total ( concha )

∫ dA ⇒ B(2πr ) = µ 0

π ( R3 − R2 ) S 2

2

2 I ( concha ) ⎛ 1 2 ⎜ r Bconcha (2πr ) = µ 0 Total 2 2 ⎜ ( R3 − R2 ) ⎝ 2

r

R2

I Total ( concha )

π ( R3 − R2 ) ∫R 2

r

2

2πrdr 1

⎞ ⎟ ⎟ ⎠

Evaluando la expresión anterior, resulta:

r µ 0 I Total ( concha ) r 2 − R2 2 ( )(−uˆ t ) Bconcha = 2 2 r 2π ( R3 − R2 ) 2011 

Enrique Rubio  http://guiasdeejercicios.blogspot.com  Evita imprimir este documento si no es necesario. Si requieres imprimir una parte hazlo  en hojas de reciclaje. Estarás salvando al planeta 

42

Física IV. Guía de Problemas Resueltos  Prohibida la reproducción parcial o total de este material sin la autorización por escrito de los autores 

(Recuerde que el signo negativo se debe a que el sentido de esta corriente es contrario al anterior, el cual se había considerado como positivo. El hecho de que la corriente esté en sentidos opuestos tanto en el cilindro interior como en la concha cilíndrica, se refleja en la dirección del campo magnético)

Ahora, el campo magnético total en la región indicada, será:

r r r BTotal = BConcha + BCilindro Sustituyendo los respectivos campos, se tiene: 2 r µ0 I µ0 I r 2 − R2 BTotal = (uˆ t ) + ( )(−uˆ t ) 2 2 2πr r 2π ( R3 − R2 )

Operando sobre esta expresión, resulta, finalmente:

r µ I R 2 − r2 )(uˆt ) B( R2 ≤ r < R3 ) = 0 ( 32 2πr R3 − R2 2

4º Para

r ≥ R3

(véase Figura 2.3.4):

En esta región, la trayectoria amperiana enlazará tanto al cilindro

R3

macizo como a la concha cilíndrica. Sin embargo, dado que las

R1

corrientes que pasan por estos son iguales en magnitud, pero

r

tiene

R2

opuestos,

entonces,

el

campo

magnético

resultante será nulo. Trayectori a Amperiana

Figura 2.3.4

sentidos

r r B( r ≥ R3 ) = 0

De esta manera, la función campo magnético, será: 2011 

Enrique Rubio  http://guiasdeejercicios.blogspot.com  Evita imprimir este documento si no es necesario. Si requieres imprimir una parte hazlo  en hojas de reciclaje. Estarás salvando al planeta 

43

Física IV. Guía de Problemas Resueltos  Prohibida la reproducción parcial o total de este material sin la autorización por escrito de los autores 

µ0 I ˆ 2 r (u t ) 2πR1 µ0 I (uˆt ) 2πr 2 µ 0 I R3 − r 2 ˆ ( 2 2 )(ut ) 2πr R3 − R2

r B(r ) =

r 0

para r ≤ R1 para R1 ≤ r < R2 para R2 ≤ r < R3 para R3 ≤ r

r B

µ0 I 2πR1

µ0 I 2πR2

µ0 I 2 2πR1

r R1

R2

Graficando esta función a trozos, se tiene:

2011 

Enrique Rubio  http://guiasdeejercicios.blogspot.com  Evita imprimir este documento si no es necesario. Si requieres imprimir una parte hazlo  en hojas de reciclaje. Estarás salvando al planeta 

44

Física IV. Guía de Problemas Resueltos  Prohibida la reproducción parcial o total de este material sin la autorización por escrito de los autores 

Calcular el campo magnético en un punto p de un hueco cilíndrico de radio a que se ha

2.4

hecho en un cilindro de radio b, si la corriente total en el conductor es I y la densidad de corriente es uniforme en su sección transversal. b

a

Punto

kˆ ˆj

d a

I



b Vista Superior

SOLUCIÓN Para este problema se tendrá que emplear el Principio de Superposición. Se seguirá un procedimiento similar al seguido en el problema 3 del presente capítulo. Entonces, se separará el problema en dos partes: cálculo del campo magnético producido por el cilindro grande (sin cavidad) y, luego, el producido por el cilindro pequeño (cavidad). Posteriormente, se sumará los efectos (tomando en consideración los signos de las corrientes). Sin embargo, se deben tener presente las mismas consideraciones que se tuvieron en el problema 3 con respecto a la corriente que se utilizó. Por otra parte, es importante notar que, en este caso, el punto sobre el que se hallará el campo magnético no está en la línea que une los ejes de los cilindros. A continuación se observará como cambia esto el problema.

Punto

b

r r r (1) BTotal = BCilindro − BCavidad

ρ A través de la Ley de Ampere: Trayectoria Amperiana

Figura 2.4.1

r r r r ( 2) B ⋅ d l = µ J ⋅ dA 0∫ ∫ C

2011 

A

Enrique Rubio  http://guiasdeejercicios.blogspot.com  Evita imprimir este documento si no es necesario. Si requieres imprimir una parte hazlo  en hojas de reciclaje. Estarás salvando al planeta 

45

Física IV. Guía de Problemas Resueltos  Prohibida la reproducción parcial o total de este material sin la autorización por escrito de los autores 

1º Cilindro grande (se considerará que la distancia del centro del cilindro al punto es ρ. Véase la figura 2.4.1):

En este caso, la densidad de corriente del cilindro (recuerde que, en la realidad, esta corriente solo circula a través del área del cilindro y no de la cavidad), será:

r r I Total = ∫ J ⋅ dA ⇒ I Total = J ∫ dA ⇒ I Total = J ∫ 2πrdr A

A

A

No obstante, véase que esta área (la de integración) es el área sólo del conductor, por lo tanto, hacer la regla de tres es equivalente a restar el área del conductor sin cavidad y el área de la cavidad (esto se puede hacer gracias a que la corriente se distribuye uniformemente a través de toda el área transversal del conductor). Luego:

⇒ I Total = J ( ACilindro − Acavidad ) ⇒ I Total = J (πb 2 − πa 2 ) ( 3) r I Total 2 2 I Total = Jπ (b − a ) ⇒ J = π (b 2 − a 2 ) (Es importante darse cuenta de que éste es un procedimiento alternativo al utilizado en el problema 3 del presente capítulo. En otras palabras, es el procedimiento análogo a la regla de tres)

Sustituyendo la expresión (3) en la (2), se tiene:

µ o I Total I Total πρ ( 2 ) dA ⇒ B = dA π (b 2 − a 2 ) π (b 2 − a 2 ) ∫S S

B ∫ dl = µ 0 ∫ C

B (2πρ ) =

µ o I Total

π (b − a ) 2

2

(πρ 2 )

(Véase que el área S de la integral es el área encerrada, que viene determinada por el radio de la trayectoria amperiana. Dicha trayectoria es circular, por lo que el área encerrada por ella es el área de un círculo que se considerará, de momento, que tiene radio ρ).

2011 

Enrique Rubio  http://guiasdeejercicios.blogspot.com  Evita imprimir este documento si no es necesario. Si requieres imprimir una parte hazlo  en hojas de reciclaje. Estarás salvando al planeta 

46

Física IV. Guía de Problemas Resueltos  Prohibida la reproducción parcial o total de este material sin la autorización por escrito de los autores 

De esta integral, resulta:

r BCilindro =

µ 0 Iρ

2π (b − a ) 2

2

(uˆ ρ ) ( 4 )

(Observe que el vector unitario no es el acostumbrado, ya que el campo magnético tanto del cilindro como de la cavidad son tangenciales, pero a sus respectivas trayectorias amperianas. En otras palabras, cada una tendrá un vector unitario tangencial que no son los mismos. No obstante, posteriormente se hará una equivalencia entre ellos) 2º Cavidad (considérese que la distancia entre el centro de la cavidad al punto es r. Véase la figura 2.4.2):

La densidad de corriente que se utilizará es la misma que se halló en la parte anterior.

Por lo tanto:

I Total I Total dA B ( 2 r ) dA ⇒ π = µ 0 2 2 2 2 ∫ ( b a ) ( b a ) π − π − S S

B ∫ dl = µ 0 ∫ C

Punto

(En este caso, tal como en el anterior, la superficie S es la encerrada por la trayectoria amperiana utilizada. Entonces, esta

a

integral es equivalente al área del círculo que se considerará, de

r

momento, que tiene radio r) Trayectori a Amperiana

B (2πr ) =

Figura 2.4.2

µ 0 I Total

π (b − a ) 2

2

(πr 2 )

De esta expresión, resulta:

r BCavidad =

µ 0 I Total r (uˆ r ) ( 5 ) 2 2 2π (b − a )

2011 

Enrique Rubio  http://guiasdeejercicios.blogspot.com  Evita imprimir este documento si no es necesario. Si requieres imprimir una parte hazlo  en hojas de reciclaje. Estarás salvando al planeta 

47

Física IV. Guía de Problemas Resueltos  Prohibida la reproducción parcial o total de este material sin la autorización por escrito de los autores 

Ahora bien, si se considera que el eje del cilindro (en la dirección de la corriente) coincide con el eje Z, entonces se podrá escribir:

uˆ ρ = kˆ × ρˆ ( 6 ) Y, también:

uˆ r = kˆ × rˆ ( 7 ) Aquí, el vector



es el vector unitario en la dirección del eje Z, mientras que



y

ρˆ , son los

vectores unitarios en la línea que une al centro de la cavidad y del cilindro con el punto sobre el que se está hallando el campo magnético, respectivamente.

Por lo tanto, sustituyendo (6) en (4) y (7) en (5), resulta:

r BCilindro =

µ 0 I Total ρ ˆ (k × ρˆ ) (8) 2 2 2π (b − a )

r BCavidad =

µ 0 I Total r (kˆ × rˆ) ( 9 ) 2 2 2π (b − a )

Ahora, sustituyendo (8) y (9) en (1), resulta:

r BTotal =

2011 

[ ρ (kˆ × ρˆ ) − rˆ(kˆ × rˆ)] −a )

µ 0 I Total

2π (b

2

2

Enrique Rubio  http://guiasdeejercicios.blogspot.com  Evita imprimir este documento si no es necesario. Si requieres imprimir una parte hazlo  en hojas de reciclaje. Estarás salvando al planeta 

48

Física IV. Guía de Problemas Resueltos  Prohibida la reproducción parcial o total de este material sin la autorización por escrito de los autores 

Por propiedades del producto vectorial, la expresión superior se puede convertir en:

r r [ (kˆ × ρ ) − (kˆ × r )] −a )

µ 0 I Total

r BTotal = r BTotal

2 2π (b 2 µ 0 I Total ˆ × ( ρr − rr ) (10 ) = k 2π (b 2 − a 2 )

[

]

Ahora, véase la figura 2.4.3:

Recordando las operaciones de vectores, se puede hacer

Punto

r

ρ

r r

r d Figura 2.4.3

la siguiente representación:

r r r d = ρ − r (11) Sustituyendo la expresión (11) en la ecuación (10), se tiene:

r BTotal =

r [ kˆ × d ] −a )

µ 0 I Total

2π (b 2

2

Haga el producto cruz con la regla de la mano derecha y, resultará, finalmente:

r BTotal =

(Donde

2011 

µ 0 I Total d (uˆ r ) 2π (b 2 − a 2 )

uˆ r

es el vector radial al cilindro)

Enrique Rubio  http://guiasdeejercicios.blogspot.com  Evita imprimir este documento si no es necesario. Si requieres imprimir una parte hazlo  en hojas de reciclaje. Estarás salvando al planeta 

49

Física IV. Guía de Problemas Resueltos  Prohibida la reproducción parcial o total de este material sin la autorización por escrito de los autores 

2.5

Calcule el flujo magnético por unidad de longitud a través de una superficie plana (S) dentro de un alambre, tal como se muestra en la figura.

I

R

S

SOLUCIÓN Para calcular el flujo magnético a través de una superficie determinada, se debe recurrir a la fórmula:

r r (1) Φ Magnético = ∫ B ⋅ dA S

R

r B I'

La corriente I’ que se muestra en la figura es una línea de corriente que se encuentra dentro del alambre (por el cual

r dA

r

circula la corriente total I). Ahora, para el diferencial de área que se está considerando, el campo magnético tiene la

L

misma dirección. Sin embargo, dicho campo, producido por la línea de corriente, depende de la distancia del punto a la fuente y, en este caso, los puntos que forman parte de la

dr

superficie se encuentran a distancias distintas de la fuente

Figura 2.5.1

en cuestión.

Luego, el área de la banda (diferencial de área) se puede escribir como:

dA = Ldr ( 2 )

Ahora, para hallar el campo magnético, se empleará la Ley de Ampere:

r r r r ( 3) B ⋅ d l = µ J ⋅ d A ⇒ B dl = µ JdA 0∫ 0∫ ∫ ∫ C

2011 

S

C

S

Enrique Rubio  http://guiasdeejercicios.blogspot.com  Evita imprimir este documento si no es necesario. Si requieres imprimir una parte hazlo  en hojas de reciclaje. Estarás salvando al planeta 

50

Física IV. Guía de Problemas Resueltos  Prohibida la reproducción parcial o total de este material sin la autorización por escrito de los autores 

La densidad de corriente, dado que la corriente se distribuye uniformemente a través de toda el área transversal del cilindro, se puede escribir, sin hacer los cálculos de los problemas anteriores, como: ( 4) r I J = 2 πR

Sustituyendo la ecuación (4) en la ecuación (3), se tiene:

µI I dA ⇒ B(2πr ) = 0 2 (πr 2 ) 2 πR πR S

B(2πr ) = µ 0 ∫

(Véase que el área de la integral, S, es el área encerrada por la trayectoria amperiana. Se debe recordar que la línea de corriente representa, en realidad, la corriente que circula a través de toda el área de este círculo descrito)

De esta forma, resulta:

r µ 0 Ir ( 5 ) B= 2πR 2

Sustituyendo las ecuaciones (5) y (2) en la expresión (1) (se debe recordar que la dirección del campo magnético es la misma dirección que la del diferencial de área. Por lo tanto, el producto punto de sus vectores directores es 1), se tiene:

Φ Magnético = ∫

R

0

µ0 I µ LI 1 rLdr ⇒ Φ Magnético = 0 2 ( r 2 ) 2 2πR 2πR 2 0 R

Evaluando esta expresión, resultará, finalmente:

Φ Magnético L

=

µ0 I 4π

(Este resultado es el flujo magnético por unidad de longitud del alambre) 2011 

Enrique Rubio  http://guiasdeejercicios.blogspot.com  Evita imprimir este documento si no es necesario. Si requieres imprimir una parte hazlo  en hojas de reciclaje. Estarás salvando al planeta 

51

Física IV. Guía de Problemas Resueltos  Prohibida la reproducción parcial o total de este material sin la autorización por escrito de los autores 

2.6

Se tiene un cable coaxial, largo, formado por un conductor cilíndrico macizo de radio a que se encuentra dentro de un cilindro de grosor despreciable de radio b. Por el conductor interno circula una densidad de corriente que depende en cada punto de la distancia r del punto al eje del cilindro de la forma:

Donde

r C J = kˆ r

kˆ es un vector unitario en la dirección del eje del cilindro. Por la concha externa de

radio b circula una corriente I1 distribuida uniformemente en toda su superficie y en dirección contraria a la que circula por el cilindro interior.

Z 2.6.1

Calcule el campo magnético inducido en los dos cilindros coaxiales que se muestran en la figura en todos los puntos del espacio.

2.6.2

Un hilo de longitud L, por el que circula una corriente I3, se coloca en el plano XY, en el

Y

sistema de coordenadas mostrado en la figura. El hilo es paralelo al eje Y y su punto medio se encuentra a una distancia d del eje del cable.

X

¿Cuál es la fuerza total sobre la barra de longitud L?

L SOLUCIÓN 2.6.1

Para calcular el campo magnético producido por el cilindro, se debe separar el análisis en tres regiones, ellas son:

1. 2. 3.

2011 

r≤a ab

Enrique Rubio  http://guiasdeejercicios.blogspot.com  Evita imprimir este documento si no es necesario. Si requieres imprimir una parte hazlo  en hojas de reciclaje. Estarás salvando al planeta 

52

Física IV. Guía de Problemas Resueltos  Prohibida la reproducción parcial o total de este material sin la autorización por escrito de los autores 

Ahora, empleando la Ley de Ampere, se tiene:

r r r r (1) B ⋅ d l = µ J ⋅ dA 0∫ ∫ C

A

1º Para

r≤a

(el radio de la trayectoria amperiana es menor que el del cilindro interior de radio a):

Sustituyendo la expresión de la densidad de la corriente en la ecuación (1) (en este caso, a diferencia de los problemas anteriores, la densidad de corriente no es constante. En otras palabras, la corriente no se distribuye uniformemente a través de toda el área transversal del cilindro interior), se tiene:

Z r J

C ˆ k ⋅ dAkˆ r S

B ∫ dl = µ 0 ∫

r dA

C

r

Trayectori a Amperiana

⇒ B(2πr ) = µ 0 C ∫ o

a

⇒ B(2πr ) = 2πµ 0 C (r 0 ) r

Corte Transversa l

Evaluando

Figura 2.6.1

2º Para

2πr dr r

a≤r